2
$\begingroup$

I am studying statistical physics for an exam scheduled next week and there's something I really do not get about mean field approximation in the Ising model.

The situation

In the lesson, we defined the Ising model as $N$ spins $S_i \in \{-1,1\}$ ; we consider the case where $B=0$ (no external magnetic field), and thus the Hamiltonian of the system is : $$H = -J \sum\limits_{i,j \: neighbors} S_i S_j$$

The teacher then said "this is an $N$-body problem which we cannot solve ; let's make an approximation to have $N$ independent $1$-body problems instead. In $H$, the term in $S_i$ is : $$-S_i \left( J \sum\limits_{j \: neighbors \: of \: i} S_j \right)$$ We replace the $S_j$ by the magnetization $m= \frac{1}{N} \sum\limits_i \langle S_i \rangle$ : $$-S_i \left( J \sum\limits_{j \: neighbors \: of \: i} m \right)$$ Assuming the number of neighbors of any two spins to be the same, that we denote $z$, we thus get: $$-S_i \times Jzm $$ Defining $h = Jzm$, we get: $$-S_i \times h$$ Which is the Hamiltonian of a spin in a field $h$. Therefore, we can apply statistical physics (canonical ensemble) and it follows that: $$\langle S_i \rangle = \text{tanh}(\beta h)$$ This result being independent of $i$, we can sum it over all possible $i$ and divide it by $N$, which yields: $$m = \text{tanh}(\beta J z m)$$ "

My problem

I don't understand why we say that the Hamiltonian for $S_i$ is $-S_i \left( J \sum\limits_{j \: neighbors \: of \: i} S_j \right)$ and not $-\frac{1}{2}S_i \left( J \sum\limits_{j \: neighbors \: of \: i} S_j \right)$. What leads me to think of this are the following arguments:

  1. By using the professor's formula, we give all the energy of the interaction between two neighbors spins $i$ and $j$ to spin $i$, leaving no energy for spin $j$. But then, we proceed on summing over all possible spins, so we will sum counting each energy twice: once for $i$ and once for $j$.
  2. The basic idea is to transform our $N$-body problem into $N$ $1$-body independent problems. To do such thing, I could start by saying that $H$ the Hamiltonian of the $N$-body problem must be the sum of the $N$ $1$-body problems, because these $1$-body problems are independent. So, something like : $$H = \sum\limits_{i=1}^N H_i$$ Back to our situation, I notice that: $$H = -J \sum\limits_{i,j \: neighbors} S_i S_j = \sum\limits_i - \frac{1}{2} S_i \left( J \sum\limits_{j \: neighbors \: of \: i} m \right)$$ So my factor $1/2$ arises naturally from not counting twice the same pair of spins (i.e. summing the $(i,j)$ term and after that summing it again by naming it $(j,i)$).

But I must be wrong somewhere, because doing what I propose would lead to the equation: $$m = \text{tanh}(\beta \frac{J}{2} z m)$$ Instead of the correct equation (for what it's worth, I know that it is correct "for sure" because Wikipedia says so, had it been wrong someone would have fixed such a big Wikipedia page already): $$m = \text{tanh}(\beta J z m)$$

Thanks a lot in advance for your help, that'd be greatly appreciated - and happy new year !

$\endgroup$

1 Answer 1

1
$\begingroup$

The starting point, $$H = -J \sum\limits_{i,j \: neighbors} S_i S_j \tag{1}$$ is sometimes used in the textbooks, but, without an explicit indication of what is meant by summing on $i,j \: neighbors$ is misleading.

If the pair interaction between two neighbor spins is $-JS_i S_j$, the formula $(1)$ should be intended as $$H = -J \sum\limits_{i<j \: restricted \: to \: nearest \:neighbors} S_i S_j, \tag{2}$$ that can always be rewritten as $$H = -\frac{J}{2} \sum\limits_{i,j \: restricted \: to \: nearest \:neighbors} S_i S_j. \tag{3}$$ These forms do not hamper to get the correct equation for $m$ in the absence od an external field. The most convincing way to see it, in my opinion goes as follows:

  1. Introduce the average magnetization and introduce a fluctuation $\delta S_i$ such that $S_i=m+\delta S_i$.
  2. Rewrite the product $S_iS_j$ neglecting the $\delta S_i \delta S_j$ term: $$m^2 + m (\delta S_i + \delta S_j)$$.
  3. add and subtract an $m^2$ term to recover the original $S_k$ degrees of freedom: $m(S_i + S_j) -m^2$.
  4. Use the previous approximate expression in place of $S_i S_j$ in formula $(3)$.

You'll end up with the following mean-field Hamiltonian: $$H = -\frac{J}{2} \sum\limits_{i,j \: restricted \: to \: nearest \:neighbors} [ m(S_i + S_j) -m^2 ]. \tag{4}$$ The second term in the square bracket does not depend on the summation indices. Therefore, when summed, it provides a spin-independent term in the Hamiltonian equal to $$ \frac{J}{2}m^2Nz $$ because $Nz$ is the number of the ordered nearest neighbors pairs. The factor $\frac12$ correctly counts the independent ones.

In the remaining sum, the role of $i$ and $j$ is symmetric. Therefore, by renaming the indices in one of the two summations, we get twice one of them: $$ -\frac{J}{2} \sum\limits_{i,j \: restricted \: to \: nearest \:neighbors} m(S_i + S_j) = -J m \sum\limits_{i,j \: restricted \: to \: nearest \:neighbors} S_i = -J m z \sum\limits_{i } S_i . $$ In the last passage, I have used the independence on $j$ to get a factor equal to the number of first neighbors $z$. The final form of the equation $(4)$ is $$ H= \frac{J}{2}m^2Nz -J m z \sum\limits_{i } S_i. $$ From this expression for the mean-field Hamiltonian, the usual equation for $m$ can be recovered.

Notice that the spin-independent term is necessary both to get the equation for $m$ from the minimum condition on the free-energy and show that below the critical point, the solution $m=0$ is not stable below the critical point.

$\endgroup$
1
  • $\begingroup$ Thanks a lot for that fast answer ! I understood how the derivation works. My mistake was to not take into account the fact that expanding the $S_j$ around $m$ implied doing so with $S_i$ as well, therefore giving the missing $2$ factor. That's clear now. Thanks ! $\endgroup$
    – Kal8578
    Commented Jan 1, 2022 at 19:23

Your Answer

By clicking “Post Your Answer”, you agree to our terms of service and acknowledge you have read our privacy policy.

Not the answer you're looking for? Browse other questions tagged or ask your own question.